• Contact Us
  • Student Login
  • My Cart

LSAT and Law School Admissions Blog

You are here: Home / LSAT Prep / Need LSAT Advice? How to Help Us, Help You

August 8, 2016

Need LSAT Advice? How to Help Us, Help You

Need LSAT Advice? How to Help Us, Help You

One of the most common LSAT preparation questions we receive is: “I’ve been studying for a while and want to increase my score even more. Do you have any tips to help me out?” This is a  question we love to get because it directly relates to what we do. We love the LSAT, and we are passionate about helping students get better at taking this test.

When answering this question, one of our goals is to provide advice that is specific to the student. While it would be easy to put out a blanket response, that type of answer isn’t all that helpful. But, to provide a personalized answer we need a lot of specific and detailed information. So, if you are asking us a question, what info should you provide?

Test Date

The first thing we need to know is when you are taking the LSAT. That tells us how much time you have to prepare, and will allow us to best shape our overall advice.

Study History

It is important for us to know how long you’ve been studying. The advice we give to a student who has been studying three months is going to be different than the advice we give to one who has been studying for three weeks. And, not only do we need to know how long you’ve been studying, but also how many hours a week you’ve been able to study on average. A month of studying 4 hours a week is a lot different than a month of studying 20 hours a week!

Scoring History

If you’ve taken any practice tests, it’s extremely helpful for us to know how well you did. And, if you use our free LSAT scoring system on our Self-Study Site or Testing and Analytics Platform to thoroughly review your practice tests, pass along your score results to us. Those score reports  give us a unique and incisive view into your performance, and allow us to see patterns that can help us give you very specific and helpful advice. However, even if you don’t use our scoring system (and you should!), give us a breakdown of how you generally score in each section, including how many questions you typically finish, and the type of variation you see in scores.

Target Score

Even if your goal is 180 (and why not?), let us know your target score. The increase you need to achieve your goal can affect the advice we give.

Preparation Material

We also need to know what LSAT preparation material you’ve been using, and what material you have on hand. For example, if I know you have the Logical Reasoning Bible, that allows me to reference specific chapters or pages for you to review. Or, if I know you have a Princeton Review book, I can tell you to start using it as a doorstop for now and then to set it on fire when it gets cold later this year 😉

Seriously, when we know what you’ve been studying, we know what knowledge you’ve obtained, what you should work on, and where to direct you for information. And don’t worry if you are using materials or were in a course that isn’t from PowerScore. We know that there are other materials and courses out there, and we realize that students sometimes start elsewhere. That’s ok—just give us all the details you can.

One side note: If you are looking for tips in one of the three sections and haven’t yet read our LSAT Bibles, that is most likely where I will refer you for advice. Those books are packed with hundreds of tips, techniques, methods, and strategies, and they collectively span almost 2000 pages. There’s no way I can distill all of that into a few short paragraphs! While I can comment on things you should change or areas you should study, if you ask me about fundamentals—such as how to solve an Assumption question—the Bibles are the gold standard for that type of information.

Problem Areas

One thing we ask all students to do is keep a log of questions missed and the problems encountered while studying. When assessing how to best increase your score, it is immensely helpful to know what you see as your most difficult question types, and also the patterns you have seen within your test taking. We can often see connections that you might otherwise miss.

As you might have noticed, the key is to give us as much information as possible. The basic rule I have is that if you think it might be useful, then include it! Our goal is to give very specific, tailored advice to each student and the more information we have about your strengths and weaknesses, habits and patterns, fears and preferences, the better.

Have any questions or comments? Please post them below!

FacebookTweetPinEmail

Posted by Dave Killoran / LSAT Prep / LSAT Prep 32 Comments

  • Rebecca Schultz
    August 08, 2016 at 4:10pm

    Hi! My name is Rebecca, and I currently would like to get into GW law and UVA. For GW I would really like a score of 166, and for UVA I would like a 170. I started at 150 and have improved to 163, but lately my scores have been falling instead of rising. It’s hard to pinpoint exactly what my problem is, so I would have to say it’s timing. I usually don’t get to 3 or 4 questions in each LR section, and I can only get to 3/4 of the RC passages. I usually perform well on the games section. I am starting to feel a little burned out from this test because I am not seeing further improvement. I will be taking my test Sept. 24, 2016, so in a little over a month. When I review, I often rescore based on what I learned, and I realize that I miss many questions that I should have gotten right. This is always a problem for me, and perhaps I am just not good with Standardized tests. I want to improve my actual LSATs to the scores I get after revisions. What should I be working on right now?
    Thanks,
    Becca

    • Dave Killoran
      August 08, 2016 at 5:42pm

      Hi Rebecca,

      Thanks for the information! One question–can you tell me what preparation books and materials you have, and whether you’ve taken an LSAT course or used a tutor? That will help me direct you better. Thanks!

  • Nancy
    August 08, 2016 at 6:20pm

    helloo,

    I have taken my LSAT in June of 2016, I am now trying to retake my exam in September but I do not know how to go about studying again. I scored a 146 the first time and reviewing my answers my nervousness really did get it me. I am aiming for around a 160, I was consistently scoring in the 150-152 range. I have taken a kaplan prep course but I want to find a different method of studying or any study schedules you make have in mind. I would apperciate the help. Also, I have all the powerscore bibles as well.

  • Dave Killoran
    August 08, 2016 at 7:00pm

    Hi Nancy,

    Thanks for the question! Can you tell me a bit more about how you do on each section? For example, is it LG that you find hardest, or perhaps LR? And how many right do you get in each section, and so on. AS much info as you can provide would be helpful!

    We provide free study plans for the LSAT Bibles and I can make suggestions based on those depending on how you are scoring in each section.

    I look forward to hearing back from you. Thanks!

  • Rebecca Schultz
    August 08, 2016 at 7:09pm

    Hi Dave, I have all three powerscore bibles. I am actually working on Chapter 12, Resolve the Paradox now. I completed the LG Bible and have only done the first few chapters of RC. I take a preptest every week and have used the LSAC 3 tests and explanations book as well.
    Thanks.

  • Christina Yates Lillie
    August 08, 2016 at 7:17pm

    On a logical reasoning question that begins with a source name, are we supposed to make an inference based on the source regarding their reliability? For example, a doctor or researcher may be considered more reliable or a better source and an advertisement or essay?

  • Dave Killoran
    August 08, 2016 at 7:51pm

    Hi Christina,

    I actually talk about how this all works int he LSAT Logical Reasoning Bible (and in our courses too, of course), but this article I wrote might help make the basic idea clearer.

    Please let me know if that helps. Thanks!

  • Dave Killoran
    August 08, 2016 at 8:22pm

    Hi Rebecca,

    Thanks for the additional information! First, you *need* to take some time off. The burnout comment is always a red flag, and besides, time away from the test can be immensely helpful. While you may feel you don’t have time to take a break, trust me when I tell you that without one, the situation will likely get worse.

    Second, it appears that you still have a ways to go in the LRB, which is a good thing. LSAT score increases are very rarely linear, and it takes time for the information to settle in. Thus, score increases are not a straight line–they tend to move from plateau to plateau. But it can be quite frustrating to be at one plateau and be waiting to move up again! So, what you describe is actually quite normal, but it sounds like it is starting to have a negative mental effect on you.

    Third, that negativity is starting to come through in how you talk about the test, and if you follow my writings, you know I am a firm believer that how you feel about this test will predict how you do on the test. So, at the moment I’d say you are fighting yourself as much as you are fighting the LSAT. Before you take a few days off, look through some of our resources on Test Mentality, which are listed here. I’m hoping that those, in combination with a break, will allow you to come back feeling much stronger and fresher!

    When you come back, keep working through the LRB, but also focus on reading closely. It’s clear that you understand things in hindsight, and so you need to make sure that you understand the facts while reading questions the first time. I’d also go back through your question tracker and look for patterns in your errors. For example, is it often Flaw question that you miss? If so, you can attack those and work with abstract language to get better at recognizing what LSAC is doing.

    The other thing is that you need to begin focusing on RC. Doing only 3 passages is really hurting. Can you describe your approach to the passages? for example, how long does it take you to read the passage? And do you mark or underline it heavily? do you return to the passage frequently when answering the questions? How frequently? There well may be adjustments that are needed there.

    That’s a start, but as you can see, each answer opens up a whole new line of inquiry and discussion 🙂 Thanks!

  • Rebecca Schultz
    August 08, 2016 at 9:59pm

    Thanks Dave, I am planning on vacation from the 13th-21st. Should I start break then and try to squeeze in as much practice now or start break now? With RC, I usually take a while to read, not sure exactly how long, but to do one entire passage and answer all the questions I can take around 11 minutes. I have always been very tedious and read quite slowly. I’m not sure how to improve my time management because then I don’t think I’ll be able to digest the information enough.

  • Dave Killoran
    August 08, 2016 at 11:29pm

    Hi Rebecca,

    Yes, I’d probably start the break now. I’m not sure what you can gain over the next few days while you are feeling burned out. But, as you go on break, review the test mentality resources I linked earlier. The recorded seminar in particular will help!

    You should time yourself on your reading of the passage only. It would be useful to know, and there are different ways to experiment with reading. The key isn’t to absorb all of the information on your first read, but to know where you can find all of the information. No one can remember every detail and so you have to return to the passage. If you know where the info is, that speeds up the process. It’s an approach I’ve used with other students to great effect, and it’s deeper than can be outlined here of course, but it would represent a shift in your strategic thinking. You can’t put it into effect until you know how much time you are taking to read a typical passage however!

    Thanks!

  • Rebecca Schultz
    August 10, 2016 at 7:49pm

    Thank you Dave. I think you’re right in how I need to improve my RC. I realize now that I do try to retain all of the information so that I won’t need to look back, especially when the topic is somewhat science-related or has a lot of different views. I started incorporating some of the passage-writing taught by the RC Bible, such as writing ‘D’ next to a definition or circling a new viewpoint. I will time myself on RC passages and let you know what I find. Also, should I be worried that I haven’t done any recent tests yet? I haven’t worked with comparative reading yet, so I am hoping that won’t negatively affect my score. I will read those test mentality resources you linked during my break. Thanks again for the advice.

  • Dave Killoran
    August 11, 2016 at 6:43pm

    Hi Rebecca,

    It’s not an issue yet that you haven’t been focused on the newer test just yet. Once you start back up, however, let’s work those more recent exams into your preparation schedule.

    As far as Comparative Reading passages, there are some differences from “regular” RC passages, but overall the same basic skills are tested in each passage type. And, many students actually like the CR passages more since it’s two different takes on the topic at hand, and the fact that each passage is shorter often makes them feel more easily digestible.

    So, you are still in great shape at this point, and it’s not a problem that you haven’t worked the newer LSATs in yet. Once you come back, start adding them in and you’ll be good. Thanks!

  • Ruthie Campbell
    August 17, 2016 at 5:52am

    Hello Dave,
    My name is Ruthie. I have the taken the February 2016 LSAT and plan to retake the LSAT in December. I’m currently enrolled in Powerscore full length course. I’m about 8 weeks into the course. My questions are for 1) should I start taking practice exams now, 2) after the full length course is over should I take an advanced logic games or logic reason online course , 3) what can I do to increase my chances of scoring an 180. I study an average 20 hours per week.

  • Dave Killoran
    August 17, 2016 at 4:33pm

    Hi Ruthie,

    Thanks for the question! Let me see if I can help you out here:

    1. Yes, you should 🙂 Here are some tips for doing that.

    2. That depends. Can you let me know how you are performing in those sections as well as the type of questions you are missing? That would help me give you the best advice possible!

    3. Ok, in the blog above, I outline the type of information that would be helpful when assessing how someone should study in order to improve. So, go back through the blog and then post that type of info for me if you can 🙂 I will say that there is no way to guarantee a 180. At that level, even a slight wobble can torpedo your efforts, and there is hardly any room at all for mistakes. But, I’ve worked with multiple students who have scored a 180, so I know the traits they have that are necessary to making that happen. Give me some more information, and we can talk about it in more detail.

    I look forward to hearing back from you. Thanks!

  • Rebecca Schultz
    September 12, 2016 at 3:32pm

    Hi Dave,
    I have been super busy since starting back at school. I am taking 6 classes and working part time as well as staying active in my organizations. I am nervous about the upcoming LSAT on the 24th because over the summer I had been consistently scoring in the 160s (highest was 163). Honestly, I have not been able to get another 163 or higher since the middle of June. Last week, I tested at 156 after a long week, and I am worried. I’m wondering why have I not been performing as well as I was since the middle of summer? Why did I start plateauing and is there anything I can do about it now? I wanted to score in the 170s (of course), but I’m not sure I can do that with only two weeks left and a full schedule, but I want to try. My RC passages usually take me about 4-5 minutes to read, so I know I am a slow reader. Should I do anything to make myself read faster? I get 10/11 wrong on RC usually, which is my worst section. Or should I study for LR? I usually get anywhere between 3-9 wrong on LR sections. I’m sorry for throwing so much into this post. I’m mostly wondering how to keep prepping at this point. Realistically, I don’t think I can do two whole practice tests a week. Let me know what you think and if I should change anything please.
    Thanks so much,
    Rebecca

  • Dave Killoran
    September 13, 2016 at 11:23pm

    Hi Rebecca,

    The first thing you should do is go back to the Test Mentality blog I linked above (here it is again) and read through those articles and watch that free test mentality seminar I did (if you haven’t already). I can see that the scoring situation is getting to you, and I want to try to put you at a greater ease before the actual LSAT arrives. Your mental outlook needs to be as calm and as focused as possible, so we need to get that in control first! 🙂

    When it comes to plateaus, the nature of the LSAT content and the test format itself makes that somewhat likely to occur regularly (I talk a bit about the phenomenon here. almost every LSAT test taker plateaus at some point in their preparation. In your case, I can’t be certain exactly why it occurred—I’d have to do questions with you to see how you think and what your strengths are as far as concept knowledge, methods, etc. But I do suspect strongly that your busy school schedule is having an impact. You’ve got 6 classes, a job, and are active in your organizations? When you add the LSAT on top of that, there’s no time to breathe or just take things in. It seems like a lot of stress and fatigue, and those two things tend to really affect LSAT learning and performance.

    Turning to your specific questions, you can’t increase your reading speed in such a short time, and forcing yourself to go faster usually just causes one to miss more questions. The thing we often talk about is not reading faster, but smarter. Focus on anticipating where the questions will come from, focus on getting down the structure of the passage and the details, and so on. By the way, RC passages are like giant LR questions to me, so the fact that you can do well on LR means that you can do well on RC too. It’s just more structure-based than LR, and getting that structure down will help you overcome the slower reading pace.

    With the time we have left, don’t worry about practice tests. What I’d do is first focus on your mentality. you need a break from this relentless pressure, and you need a change of mindset asap. So, I’d take some time to read through those test mentality articles linked above if you haven’t already. They are actually really interesting (I think!), and a nice relaxing read, and they can help re-focus how you are thinking about this test. that is more important than anything right now because I sense the tension in you when you describe what’s happening. Second, instead of doing more questions or tests, which won’t really get you to a new level, I’d go back and in a low-key, relaxed fashion look at the composition of each section and make a mental plan of what you want to do. These aren’t rules, but more a mental gameplan of how you intend to unlock each section and the questions therein. For example, in LR, go back and review argument pieces such as premises and conclusions. Look at the various question types and think a out how the information flows in each. Review questions you have done already and look at how the test makers constructed the questions, and what parts of the stimulus lead to the correct answer in each case. Establish to your own satisfaction that you know how to solve questions such as Assumptions and Flaw, and so on. Give yourself the opportunity to review everything while at the same time removing the pressure of relentlessly testing yourself. I think taking these two steps in the final days will help you more than more tests or more practice problems. You need a break, and this is the best way to get one while still studying 🙂

    Final note: the above advice isn’t for everyone, but it is good if you are in Rebecca’s situation. Before making any plans to radically change your approach, evaluate your own situation and scoring level. For some students it’s by far the best to take as many PTs as possible but for others that’s not the right approach. Choose the approach that makes you feel the most confident.

    Rebecca, please let me know if the above helps. Thanks!

  • A.R
    September 15, 2016 at 2:14pm

    Hi Dave, thanks for replying on the other post. I followed your link here and have provided the info below.
    Test Date: Dec 3rd 2016

    Study History: Since May have been taken a free prep class through University’s Law faculty for “low income, high potential” candidates. Helped me improve my diagnostic from 140 to low 150s. I’ve done a total of 9 timed PT in the PT30-50 range and some random sections on the side (definetly no more than 20 PT have been touched in total). During the summer months totalled about an average of 20-30 hrs a week of independent studying.

    Scoring History: Diagnostic 140. Other scores have been, 150,143, 151, 148, 151, 151, 155, 151, 149. Section scores are as follows: LR 7-10 wrong, Games 2-4 wrong, RC 7-10 wrong rough averages. In terms of finishing, LR most of the time i dont get to finish the last 5 and i guess, Games im typically finishing unless there is a really tricky game or i make a stupid mistake, and then RC i always guess the entire last passage alot of the times im just skimming the actual text because time is running out. Highest LR 6 wrong, Games 2 wrong, RC 7 wrong.

    Target Score: 165! but 180? sure why not 🙂

    Prep Material: All three bibles (RC is 2015 edition) and every LSAT up to PT74. Money is tight for me so i cant really afford to buy more material. I have read the RC and LR bibile not the games bible. To be honest law school students have been extremely nice and have given me some of their old LSAT prep books like Kaplan and Manhattan but havent even open them (my loyalty is to Powerscore, i hear they’re the best :))

    Problem Areas: LR is where i really want to improve as its 50% of the exam and im not really finishing it on time. Also my timing as im not completly most sections. In terms of questions on LR my weakest are probably Flawed Reasoning, Assumption, Must be True, and Weaken but i havent really been tracking my issues.

    Other info: my exam is in the morning and im a morning person so im usually in the library studying in the morning. Im back at school now 4th and final year. 10 hrs a week of class this semester in Pol Sci, so alot of reading. I really just need a set schedule or routine to follow on a weekly basis using the material i have right now. How many PT a week? Reviewing day of writing or the next day? can i split my day into lsat work in the first half of the day — lunch/break–school homework in the evening?

    Thanks alot any advice is appreciate
    Arman

  • Vanessa Davis
    January 08, 2017 at 8:45pm

    I obtained BS in 2013, now I’m ready to start my law career. I’m just not sure where to start or how to go about things. What are the steps I would need to take??

  • Dave Killoran
    January 08, 2017 at 10:36pm

    Hi Vanessa,

    Thanks for posting! I’m going to assume you’ve never taken an LSAT before, and could you also let me know when you plan on taking the LSAT? these things make a big difference when we give advice 🙂

    If you haven’t taken an LSAT before the first step is to sit down and take one; your results will tell us a lot about your strengths and weaknesses, which will in turn help me know exactly where you should start!

    To obtain a free LSAT to take, simply download the LSAT here: http://lsac.org/free-sample-test/, then sit down and take that as four consecutive 35-minute sections. To score the test, you can go here: http://students.powerscore.com/self-study/index.cfm. That will produce a readout that breaks down your performance and allows us to get an excellent view of what you need to work on.

    That’s the best place to start , but if you want to skip that step, I’d recommend buying the LSAT Logic Games Bible and the LSAT Logical Reasoning Bible (both can be found at: https://www.powerscore.com/lsat/publications/#lsatprep). Those two books cover the two sections of the test many students find most difficult.

    Please let me know what you think. Thanks!

  • Vanessa Davis
    January 08, 2017 at 10:43pm

    Your correct I haven’t taken the LSATs as of yet. I will download the LSAT and take it. Thank you so much, I’ll be in touch!!

  • james rand
    February 08, 2017 at 1:36am

    been studying since summertime went through lr powerscore cover to cover and the workshop except for that last test. 2months ago took preptest 72 timed never seen b4, got 9 wrong on comprehension and 12 wrong out of 50 or both section on the logic reasoning been doing lr take or leave it which means try to prephrase then go to a if i pass a go to b etc cant go back to choose if i moved past that choice if i get to e gotta chose that if its not e i get it wrong have gotten that down to 3 wrong per lr section and doing comprehension untimed did a tutoring session with guy i took the course with seemed very helpful used to have times where i could get 7 wrong untimed and not know why its wrong but its been 0-2 wrong since but ill still have days like today where i get 5 wrong . i wanna get those sections to zero wrong obviously but rc 4 wrong and lr 2 AND 2 would be good

  • Dave Killoran
    February 08, 2017 at 4:40am

    Hi James,

    Thanks for posting this information. If I understand you correctly, you have reached a stage where you are often 0-2 wrong in both LR sections, but there are times when it balloons up to 5 wrong, and you want to get it so that more or less never happens.

    First, it’s important to know that some variation is expected, and almost impossible to avoid. I talk about why that is the case here. And the makers of the LSAT know this as well, which is why they use score bands (see the second-to-last paragraph here). Score bands are a direct admission that your LSAT score, and by extension your LSAT performance, is expected to mover around a bit. Its’ also the case that the way the LSAT is constructed serves to affect the difficulty in each individual section. For more on how that works, see our LSAT Casino post, Question-by-Question Difficulty post, and Individual Question Difficulty post. The bottom line from all these points is that having a perfectly consistent test performance in an individual section where you are always in the same range is very tough (not impossible, just difficult). The benefit of understanding that is that you might be less frustrated when you see those score movements in the future.

    Second, when you are trying to close out those last few questions, it’s about details. Certainly the details in each question matter tremendously, but also the details in how you have been studying. How closely have you been tracking your misses, and what have you seen as far as patterns? Is there are a particular question type that trips you up? Reasoning type? A certain spot in the section (at the end, for example)? Studying your practice tests is incredibly valuable, and hopefully you have a spreadsheet that shows you every question you missed and why you missed it. If not, start making one right now! Seeing your errors over time is one of the best ways to eliminate misses since it helps you see the bigger patterns within your mistakes.

    Third, go back through your early tests and look again at you performance in each section, including both the correct ans incorrect problems. For the ones you answered correctly, could you do it faster now, and is there anything further to learn from the question? For the missed ones, would you miss those again today, and if not, why not? what have you learned since you last did that test that makes a difference? Improvement at this point isn’t going to be about big concepts or ideas, but instead about the little things, like you understanding of a certain idea or the variations the test makers use in phrasing. Look for things like that as you analyze your older tests.

    Last, have you thought about working more with a tutor to smooth out those last few questions, or perhaps about our Advanced LR Course? Tutors can be great at fixing problems, and the course is helpful for seeing some of the higher level concepts in a clearer light. Given that the questions you miss are likely tough ones, either path (or both) could be helpful.

    Please let me know if that helps. Thanks!

  • Roger Park
    April 01, 2017 at 10:33pm

    First time test taker. My test is June 12, 2017. I have the 3 bibles and the 3 workbooks. I am trying to get into U of Houston night law school. I have done absolutely nothing to prepare and haven’t been in school in over 10 years. I just decided this past week to attempt to get into law school.

    I printed the 2 month study guide and plan on starting that but will take any advice other than that to help me get near a 160 or higher score. I work full time so I am studying up to two hours in the evening and more on weekends.

  • Dave Killoran
    April 01, 2017 at 11:05pm

    Hey Roger,

    Thanks for the question! Given the timeframe and the materials, you are right to use the 2 Month Study Plan. As you use that plan, make sure that you score your exams on that same site (the scoring and performance analytics will be very helpful in helping focus your studies) and also don’t forget that we can help you with any questions you have on our LSAT Discussion forum. Those two resources will put you in a good position to identify your weaknesses and then get help when needed on conquering those.

    As far as being an older student and not having studied for years, don’t worry about that at all! It’s not as if your brain hasn’t been working at all for the last decade 🙂 You’ve been working, balancing checkbooks, reading the news, arguing with friends–all things that keep you mentally agile and fresh. You’ll find that preparing for the LSAT is kind of fun, and each question is like a mini-puzzle and challenge that keeps it interesting.

    You are already studying daily, and that’s the way to do it with this exam because you can’t cram for this test. Keep going that way and it gives you the best chance to learn the concepts and methods. Right now, your approach sounds good so keep going in that direct and check in with us when we can help. We’re here if and when you need us. Good luck and keep working hard!

  • kandice
    April 26, 2017 at 12:57am

    Hi Dave, I was not sure if you were still taking questions or not. I ran across this article back in August the day after the first post and I thought about posting then, but I find it difficult to articulate my problem well when it comes to the lsat. I have been studying for the this test for over a year. I have read the LRB and LGB. I also bought the workbooks for each and started a note book, almost like a short hand to help me remember the concepts. I started my prep with a 148 and from there I have only gotten worse. I recall getting a 150 once (probably the second or third test that I took). And that was only because I had already completed one of the logic games prior to taking the test so I breezed through it. I could probably explain to you were I am now with my progress or lack there of.

    LR- I have tendency to get Flaw, Weak, MBT wrong-not every time but often enough for it to be a pattern. I tried to target them and went back into the LRB. I also tried to drill them. I’ve noticed that I have since gotten better but I have also noticed that sometimes I have a difficult time understanding the stimulus and often have to read it twice when there is a lot of info. I try not to get distracted by the extra info like in the economic stimuli, but I still have to read it twice after reading the question. In the past, during timed sections I would only get to about 12 questions in each LR section,and then have to guess on the rest. Since then I found that I can get to maybe 19 but my accuracy suffers. Someone told me to slow down because accuracy trumps speed. When I slow down I get to about maybe 14 questions but I’ll get about 12 right and guess on the rest. Today I tried a method I think Nikki suggested to someone in the forum which was trying to do the first 10 problems in 12 minutes. I used preptest 28, but I was unsuccessful. It takes me roughly 2 minutes to answer a question, 2 1/2 to 3 minutes after the first 10 questions. Anyway, I got to 9 questions. I got 7 correct. 1 legitimately wrong. The other, I guessed wrong because I ran out of time. I noticed that I took the longest on question 4, maybe 3 minutes. I think it was a MBT X. I went through the rest timed. And I got 9 wrong in total. 3 Weak, 1 Principle Par, 2 Ass, Flaw X, MBT, MBTX. I went through it and found that I made stupid mistakes. Usually answering them too fast or dismissing the correct answer choice too quickly if it was not what I expected.

    LG- I have literally done every type of logic game. In theory, I could complete a whole section and get every answer correct but something happens when I try to time myself. I read an article which suggests doing the easier ones in under eight minutes, so that I have more time for the difficult games, but the application of this method is difficult for me. I can and have finished a game in 7 and even under 5 minutes but not consistently. This is probably the most annoying section to me because I can do the games slowly when I have time to think but under the pressure of time I think I get anxiety and I don’t really know how to get around it. I was afraid to continue doing timed section because I didn’t want to blow threw all my games if I wasn’t improving.

    RC- I have read that this is the most difficult section to improve on. At first I thought that I would be bored and zone out while reading but I have taken an interest in the news over the last few months and find myself reading news articles everyday for fun. I love history, science, and philosophy and I will gladly read it on my own. So I think my problem is not so much with the material fortunately. My problem, again I think, is that I can only get to three sections and I get an average of two wrong per passage. They are generally a mixture of author’s purpose, and passage questions, and sometimes main idea. There are some passages where I don’t get any wrong and I get it done in 10 or 11 minutes and then I jump up and down because I’m happy, but it is ultimately short lived because it is not good enough. To me it seems impossible to read the passage which takes me 2-3 minutes (depending on length and topic) and answer the questions in 5-6 minutes (depending on when I finish).

    Needless to say though, I believe my biggest issue at least in my mind is LR. It is the section I have worked the most on but yet I don’t see any improvement. I used to spend hours studying every day and taking sections but then I had to cut back because my head felt like I was slamming it against a wall. I cut back to around four hours a day but even that can be stressful. I took a few days off last week and some this week, I probably need more like a month off but the June lsat is right around the corner and I am terrified to take the extra time. I know that it can help because the lsat is a “marathon not a sprint” but I have all these what ifs’ in my head. I’ve already postpone this test entirely to many times and if I skip the June lsat my parents will think that I am crazy. I have read so many articles on the lsat it is scary ( I think I’m obsessed) but one thing I have not done is spoken to enough people about it usually because I don’t know what they could tell me that I have not already tried. However, at this point I am ready and willing to try anything. I just really believe that I can score well on this test, if I didn’t I would have quit a year ago.

  • Dave Killoran
    April 27, 2017 at 12:30am

    Hi Kandice,

    Thanks for the message! I am indeed still taking questions on this article. However, I am just leaving for our company retreat and will be gone for several days. If it’s ok with you, I’ll post a lengthy response when I’m back, probably on Monday.

    Thanks!

  • Shariful Khan
    April 27, 2017 at 6:02pm

    Hi Dave!

    So here’s the get down.

    I started off prepping for the LSAT in January. I’m taking the test in June. My initial diagnostic score was 157, and I was provided free Kaplan tutoring through the scholarship that I’m in. The Kaplan tutoring itself didn’t help me a lot but the tutor knew a lot and I was able to ask him loads of questions that helped me understand the LSAT better. I read the LSAT Trainer by Mike Kim, your Logic Games and Logical Reasoning Bibles, and have completed around 30 (?) prep tests. I still have 8 left in the 70s, which I’m waiting to use until I get closer to the actual test date.

    My goal score is a 172+. Right now, I’m scoring between 170-172. I’d like to get my practice score higher so if everything goes wrong on the day of the LSAT, I’ll still end up with a fairly high score. Through analytics, I’ve found that my worst section of LR is strengthen/weaken and flaw, but I have vastly improved that. In any case, here’s my breakdown for PT 60 and PT 75.

    PT 60:
    Score: 172
    LR: -3 (14, 21, 22)
    LG: -1 (2)
    LR: -1 (12) — super annoyed about this one, was easy in hindsight.
    RC: -3 (5, 15, 25)

    PT 75:
    Score: 170
    LR: -2 (16, 22)
    RC: -4 (14, 17, 24, 25)
    LR: -2 (8, 13)
    LG: -2 (18, 23)

    Thank you!

  • Dave Killoran
    April 27, 2017 at 6:13pm

    Ok, got it. It’s going to be a few days before I can respond due to our company retreat, but I will come back and reply in detail 🙂

  • Dave Killoran
    May 02, 2017 at 2:55am

    Hi Shariful,

    Thanks for the question! The point you’ve reached in your preparation is one where very small–almost minute–differences can have a relatively significant impact on your score. This is because in the 170s typically a single question moves you up or down a point (and sometimes two if there is a hole in the scale). Thus, it isn’t about main areas at this level but rather the long tail sections of each concept, in other words, the things that appear less frequently or that are tested more rarely. The low-hanging fruit is gone, and because of this it becomes more difficult to increase your score. I have a few thoughts here, and since they aren’t necessarily related I’ll put them in bullet point form:

    * I see you making references to various concepts and question that caused you issues, and I hope you’ve been keeping a detailed log of those mistakes and what lead to their occurrence. That is the most reliable way I know to identify the individual areas that still need work! The reason I ask is because at this level, anyone helping you needs more information than just the questions missed in order to make an insightful analysis. For example, while it helps us to know that on PT 60, LR1 you missed #14, 21, and 22, we’d need to know a lot more about why you missed them and your assessment of the causes behind the mistake. Because there are so many different reasons that lead to a missed question, to really provide the type of advice that leads to improvement requires as many details as possible.

    * Somewhat related to the prior point, are you doing the basic score analysis that we provide on our Self Study site? You can score every test there, and the stats show you which questions are difficult and which answer choices are the most attractive. It’s a trove of immensely valuable information that helps astute test takers better evaluate their own performance and also gain insight into how LSAC constructs this test. If you aren’t examining each test performance through this lens, I definitely recommend you start right away.

    * You’ve obviously done quite a bit of work, and that is awesome–congrats! You mentioned tutoring with a Kaplan instructor, and it struck me as odd that the tutoring didn’t help a lot but that the tutor knew a lot. Usually, a great tutor can help out no matter what, so that concerns me a bit in that it left you to figure out what was going on. I would bet that if you worked with a real expert–and one that was actually knowledgeable in our system–then you could get a lot further in a much shorter time. One way of looking at it is that you went to a bicycle mechanic to work on your Ferrari; sure, he could tell you a bit about the general way cars operate, but when it came to details he might as well have been advising you on how to get to the moon. It’s just a thought, but at the level you are at, and with the money on the line via scholarships from high scores, it might very well be worth the time and money to work a few hours with a new tutor. If you are interested, I can give you the names of a few PowerScore tutors I trust 100% who have worked with students in similar situations to yours.

    That’s a few thoughts to start, but you’ve come a long way already and that is amazing. You are now in the zone where small differences have a big impact, and so the needed improvements will be small things.

    Please let me know if that helps. Thanks!

  • kandice
    May 03, 2017 at 9:35pm

    Hi Dave, I hope you had a great time on your retreat. I could not help but noticed however that you did not respond to my post. I thought perhaps you were busy or simply just forgot. It’s okay. If you can’t get to this one then please don’t worry about it I’ll probably post something in the forum or something. Either way I promise I won’t bother you again. (It could just be seen as me venting). But in the time awaiting for your response I took a short break from the test. After a brief pause, I picked up my book and took a LG section from PT 63 yesterday. When I finished, I saw that I was able to answer 18 question correctly in 35 minutes (minus the answer sheet). I know it is important to practice with it but I wanted to see how many I could do when I wasn’t so stressed about that stupid sheet. It worked, I was more relaxed while taking the section. After which, I reviewed the ones that I missed. One missed opportunity was a list question which was probably due to me overlooking something. I don’t normally miss those. One was a substitution question which I was able to correct on review. It was only my second time seeing this type of question but after reviewing it I don’t think I will have further trouble with it. The last three were from the fourth game which threw me for a loop at first but after going over it I found the game to be super easy and fun. I missed 3 questions because I initially failed to make the proper inferences. For instance, #23 was super easy once I realized “W” couldn’t go in the 6th spot, but I did not have very much time to do the game. Otherwise it was a pretty simple game. Actually I found this whole section to be one of the easiest sections that I have ever taken. The third game took me the longest to complete and I got one wrong, I think it took me 11 to 12 minutes, maybe. I did them out of order, and had to rush on the 4th and the 1st game which I did in 4 minutes:) So I am far from certain, but I think my biggest issue for LG is timing. I am going to retake the third game and see how I can make inferences much faster since there were a couple of global questions in that one. I have not taken an LR section yet, I’ll probably take one tomorrow to see where I am. I took a RC section today which did not go great. I only got to three passages and I got 10 wrong but also 11 right. The first two passages I got 5 wrong on each and then absolutely nothing wrong on fourth passage. (I did them out of order). Going through it I saw that I missed 7 passage questions but just one MP (I attribute that to perhaps me rushing because I redid the passage and spotted the correct answer easily. Upon review of all of the questions I got wrong, I still had trouble with 3 passage questions and 1 author’s reference question on my second try. I’ll spend the next few weeks drilling these types of questions from the passages but I really don’t understand how to get through a passage and answer 5-8 questions correctly in eight minutes. It seems impractical. I figured it is similar to the LG section where some games take more time to do but for me I can not seem to complete a single passage in under 10 minutes. I feel like there is something I am just not getting.

  • Dave Killoran
    May 04, 2017 at 4:15pm

    Hi Kandice,

    It’s simply been a matter of me being overwhelmed by work and other obligations since I returned. You might have noticed that the recent replies I have given here have been relatively short, whereas yours will require more time for me to formulate. I am trying to get to it, but it’s been a challenge for reasons that have nothing to do with you (for example, I have family in town now, and have been playing tour guide 🙂

    You aren’t bothering me at all, and I *always* respond to every student who asks me a question. But, at the same time it can occur that I have dozens of people asking me detailed questions. Trying to answer all of those while at the same time running PowerScore can be a challenge, and thus while I strive to answer questions within a few days, that’s not always possible for me. I promise I will reply to you though, most likely on Sunday or Monday, and I apologize for that. I also definitely always recommend posting in the forum because my schedule can be variable whereas on the Forum you have other instructors answering questions. On these blogs, it’s just me for the most part 🙂

    I hope that makes sense, and thanks for your patience!

  • Dave Killoran
    May 08, 2017 at 9:20pm

    Hi Kandice,

    Thanks so much for your patience in awaiting a response for me! I know it took a while but this was a very unusual two weeks for me. For anyone else reading, if you ever have a question that is urgent or that you wish to receive a quick response to, certainly post but also post it in our LSAT Forum. The Forum is patrolled Monday through Friday as well as most weekends, and we attempt to answer questions as quickly as possible there. As I’ve said elsewhere, when questions are directed specifically at me, I try hard to provide answers within 1-2 days but on rare occasion things accumulate and it can take much longer. This was one of those situations regrettably.

    Ok, let’s move on and talk about your situation. I first want to commend you on keeping a notebook, especially one that sounds so detailed and useful. Creating and maintaining a notebook that tracks test performance is always one of my first suggestions to students, and while many people have good intentions, they end up keeping at it for a few weeks before they quit. It sounds like you’ve kept up with it, and that is admirable! If you don’t mind, can you maybe send me a snapshot of a page or two of that book, or the file of it if it is electronic? I’d like to see some of the notations you are making and get a feel for how you are analyzing your performance. Just send it to the email address of “LSAT at (@) powerscore.com” (no spaces there; I wrote it that way to avoid spambots grabbing the email and sending us tons of useless stuff) and mention that I requested it from you. I don’t expect that information to be decisive, but it will help me better understand how you view your performance, and that’s the first step towards me being able to provide you with specific, qualitative advice to break this pattern you are in.

    This is just the first part of what I’m going to post here, but I wanted to get this out to you now since I have to step away for a meeting. I’ll continue on when I return and post again later today. And just to reiterate it one more time, you aren’t bothering me at all 🙂

    Thanks and I’ll talk to you again shortly!

  • Dave Killoran
    May 09, 2017 at 12:33am

    Hi Kandice,

    I’ve been reviewing the overview of your section performance in this post and I had a quick questions: aside from the LSAT Bibles and Workbooks, your performance notebook, and then the LSATs you’ve taken, do you have any other prep materials or have you taken any courses or worked with a tutor? I have an idea in mind as to where you are running into problems, but I want to make sure I have properly understood the complete landscape of how you’ve been preparing.

    Thanks!

  • kandice
    May 09, 2017 at 3:30am

    Hi, Dave! Don’t worry, I didn’t mind waiting. I know that you’re busy, I do hope you had fun showing your relatives around:) I did send the information as you requested. To answer your question, no I have not spoken with a tutor. Not formally. If I did not understand a question or a concept I simply asked questions about the issue in the forum but that’s really it. I took an introductory seminar once back in (I think) September but since it was an intro seminar I already knew the information. I mainly just used it to ask questions with a live instructor. Before powerscore, I used a McGraw hill book for the first few months of my prep but I did not find it all that helpful. Plus the book used simulated material and provided explanations that only served to confuse the crap out of me.

  • kandice
    May 10, 2017 at 3:05pm

    Hi Dave, I forgot to mention the other day that I also study 7sage videos for my logic games and manhattanprep to review logical reasoning questions that I may have missed or if I chose an answer choice that I was unsure about.

  • Dave Killoran
    May 14, 2017 at 12:42am

    Hi Kandice,

    Thanks for all the info–I received those pics of your LSAT notebook, and that was helpful (you’ve got great handwriting by the way!). I feel that between what you’ve provided me here and the answers you have given to the questions I’ve asked that I now have a better sense of how you’ve been preparing and especially how you’ve been tracking your performance and obtaining feedback. This information is always helpful, but in your case I felt that it was essential because of your comment that, “I don’t see any improvement. I used to spend hours studying every day and taking sections but then I had to cut back because my head felt like I was slamming it against a wall.” When someone gets stuck at a certain level during studying, there are usually certain predictable reasons why and a few predictable solutions to getting unstuck as well. For example, one predictable cause is not studying enough, but from what you’ve said we can quickly eliminate that as a cause. As far as I can tell, you’ve done most all of the things you need to do when self-studying, from practicing regularly to taking multiple timed LSATs to seeking answers and feedback from various Forums and prep sources. So I don’t think the problem is coming from those quarters.

    As near as I can tell, the problems you are having are coming from a feedback loop of sorts. I suspect that when you were initially absorbing some of the foundational concept and strategy pieces you may have run into some issues, and so as you now go through self-analysis and processing explanations, those get filtered back through that initial structure, and so you end up stuck running into the same problems. That’s kind of an abstract description, but it explains why you’d go through so many explanations but then still be stuck in the same place. It’s like you’re making the same errors over and over again, and to break out of that pattern doesn’t take a small adjustment, it takes a bigger change. One of the difficulties of LSAT preparation is that the concepts and strategies are somewhat foreign to most people, and so it makes self-analysis more difficult. If you haven’t studied conditional reasoning before, then understanding why you are missing advanced problems is not easy, and it’s even harder to see how the problems you are missing might all relate to each other. In your notes I could see you quite smartly telling yourself how to avoid certain errors, but what I didn’t see (and to be fair I didn’t expect to see it) was a discussion of the abstract aspects of the reasoning behind the problem. In other words, you were talking about how you were solving the problem structurally, and what to do better there, and thus your notes addressed things like “did not check back in passage, answered from memory,” pay closer attention to question stem,” and notice the main differences between similar answer choices.” This is helpful because it sharpens your general approach to each question and makes sure you are a consistent machine in how you process answers. But, you also need a focus on the problem from a concept standpoint, and especially how the elements of a given problem might relate to other problems you’ve missed. Notations like that would sound something like, “the author ignored an alt cause; I’ve missed alt causes before, must remember that poss” or “another philosophy stim. skip these since they are always hard.” Granted, I’m only looking at a sample of your notebooks (and I like what I see!) but I’m wondering how much you’ve been able to focus on analyzing where you are missing strategies and techniques in these problems. Does that make any sense?

    Let’s assume for a moment that I’m kind in the right area with what I’m thinking (and I might not be since I haven’t had access to everything you’ve done and I haven’t had a chance to see you solve problems live). If I’m right, then probably the best way to break out of that is to work with a tutor. I suspect you’ve read a fair bit of what I’ve written on this blog and others, and you know that I don’t just blindly try to sell products to students. I’m much more interested in the perfect solution, and not the sale. I say that because tutoring is not typically a cheap solution so I’m actually a bit hesitant in most cases to recommend it (and, the general expense of LSAT preparation was one factor that lead me to write the LSAT Bibles–I wanted students to be able to access high quality instruction at a reasonable cost!). But here I’ thinking it might help because a tutor can dynamically respond to each question you have and more importantly a tutor can connect the difficulties you have to each other and see themes and patterns in what you miss. That’s what appears to be missing for you, and it’s what I would suspect could break you out of the plateau you are stuck in. You clearly work very hard, and I imagine that if you were directed correctly that hard work would start to pay dividends!

    Ok, I wanted to throw that out there so it was part of the conversation. the big piece for me is that the description of problems that you’ve provided is very good, and good enough that I feel like I’ve eliminated a number of the usual causes. I really feel like the best way to determine if a tutor could help would be for you to talk to someone for 20-30 minutes and see if their analysis (yes, even in that short of a time period) was helpful. If you want me to recommend someone specifically for you and then have to speak with them for a prelim discussion, just let me know–I’d be happy to do it.

    Last, I wanted to mention that it’s ok that your parents think you are crazy 🙂 It’s not the first time I’ve heard that, and there’s something about this test that seems to really get in your head and take over. I always loved working with LSAT-obsessed students because I knew they’d work hard at whatever I advised. So, tell your parents that you know that you might be a bit overboard but that it’s usually a sign of success. This is a big test, and a small obsession with it isn’t unusual. At the same time, it sounds like you are taking in a lot of info from different sources, and I think that’s great as long as you can keep it all ordered and in perspective. That’s something a tutor can help with too.

    That’s it for now, and I’m sorry that I haven’t been bale to reply to you faster over the last few weeks. It’s just been a really strange period of time for me and i appreciate your patience more than I can say. Thanks!

  • kandice
    May 15, 2017 at 2:58pm

    Hey Dave, thank you for responding to me. Okay, so I think I understand what you’re saying. After reading your post I went back through some of my test analysis. I found that I usually take note of the type of problems that I frequently get incorrect, yet I (shockingly) fail to pay enough attention to specific attributes. Kind of similar to seeing the forest but not the individual trees. For instance, I could get roughly two Flaw questions wrong per section and only see that I missed the (Flaw) question type. Not however, that three out of the four that I missed contained abstract language used to describe the flaw in the stimulus. And that abstract language tends to be more difficult for me to understand. Or that I don’t like inference questions because sometimes it hard for me to find a link between the premises and it often takes up my time searching for it (and that would be a skill issue). I tend to put all of my focus in deconstructing the problem not in analyzing why I missed it in the first place and then linking it to a consistent pattern. So I never get to the root of the problem, forcing me to rely on my skill of eliminating the wrong answers (which I have gotten exceedingly better at). So if this is what you are saying (if not and I am way off base here I would welcome the correction), then I trust in your judgement that it may be beneficial for me to converse with a tutor and I would be forever grateful for any recommendations you can provide.

  • Dave Killoran
    May 15, 2017 at 3:46pm

    Hi Kandice,

    Yes, that is *exactly* what I’m saying! You’ve described the questions you are missing but you haven’t gone into the root causes, and it is only by exploring those that you get better. I think this is why you’ve been stuck, and I know that’s doubly frustrating because you’ve been working hard. What we need to do is re-focus what you’ve been working on and how you’ve been studying 🙂

    Ok, I am going to speak with Maura, who heads up our Tutoring Department, and have her send you an email with my recommendations. You should hear from her later today, or tomorrow morning at the very latest!

  • AH
    August 31, 2017 at 9:00pm

    Hey Dave,

    So, here’s my story. I took the LSAT Sept. 2016 and received an average score of 152. I was not too pleased with this score, but I also knew that I am not that great with standardized tests. In preparation for that test, I was enrolled in an in-person course with TestMasters. The course was good, but I do not think I utilized all the tools fully to receive a better than average score. Even though my score was not the best, I still applied to many top 20 law schools last fall and got into some great places AND got waitlisted at some top ten schools. During that application process, I attached an addendum stating that my poor performance on the LSAT was not indicative of my academic achievements.

    All in all, I knew that I could do better, so I decided to take a year off and try again. I have been studying on and off all summer and have been hitting it pretty hard in August (utilizing old TestMasters books and the Powerscore LR Bible & LG Bible). I’ve taken 5 practice tests within the last few weeks. I scored pretty low on the first, then jumped 11 points on the second. Since my highest score, I have dropped 2-3 points each test. So now I am pretty frustrated and losing confidence. What should I do? Should I take the next week off and take another practice the week before the test (Sept. 16th)? Should I attach another addendum to these applications just in case my score doesn’t improve (I plan on submitting my applications by Sept. 15th to increase my chances)? Would attaching another addendum look like an excuse? I am honestly terrified that taking a year off did not do me well.

    I appreciate any and all help you are able to provide me.

    Thank you.

  • Dave Killoran
    September 01, 2017 at 12:13am

    Hi AH,

    Thanks for the message! There are several different parts to what you’ve asked, so I’m going to separate my reply into sections so I make sure to cover everything:

    *A Week Off

    Normally, this would be something I’d recommend, simply because breaks are not only useful but necessary along the way (see this article for more info). However, with the LSAT approaching rapidly and your score falling, in this instance we don’t have that luxury. So, it’s a good idea, but it’s just not in the cards this time.

    *Your LSAT Score

    I’d usually suggest that you go through a series of steps designed to help identify your problem areas and to help you expose the issues that are causing you difficulties (for example, along the lines of some of the ideas talked about in this article. However, given that the LSAT is only a few weeks away and that your score seems to be going back down, I’m going to recommend instead that you work with an LSAT tutor. You need to take fast action, and not only reverse the slide but also rebuild your confidence. Both those things are hard to do on your own, and with the added challenge of having very little time, a tutor seems to me to be the best and most likely pathway to success. I know it’s a financial commitment, but if it saves your scores then it would be worth it many times over 🙂 If you need any suggestions for a tutor, I could match you with one of ours who has worked with students in similar situations to yours before, with excellent results.

    *Another Addendum

    Do you mean a second addendum about scoring issues? I wouldn’t go that pathway; instead, your addendum from last year would work just fine. A second one on the same topic would indeed look like an excuse, so I wouldn’t go that route.

    *Early Applications vs LSAT Scores

    I was asked this question earlier today, and I mention it here because you seem focused on getting your application in as early as possible, but a higher LSAT score is worth *much* more than an early application. I talked about this here. If you get a chance, read through my response as well as the articles I linked–it might change your mind about your timeline, which would really help your cause here!

    Please let me know if that helps. Thanks!

  • AH
    September 05, 2017 at 4:38pm

    Could you tell me more about these tutors please?

    Additionally, I know what question types I’m consistently getting wrong. And I try to study those specific question types. And I typically do okay on practice problems but then when it comes to a practice test, all knowledge leaves my brain, unfortunately. Those include:
    -flaw questions (its mainly how the answers are worded that gets me)
    -some necessary/assumption questions
    -some sufficient/justify questions
    -some principle questions
    -some weaken questions
    Do you have any tips for those question types?

    I also have trouble with timing on the reading comp. section.

    The games are definitely my strong suit.

    Additionally, I read your article on submitting apps early vs. LSAT score. So, thank you for that.
    I believe I can score 2-4 points higher on this LSAT. But I would not be completely sure until the score is released. Should I a) press submit on my apps within the next week or so without an addendum and wait patiently for the score and hope it’s higher b) press submit on my apps within the next week or so WITH an addendum (just in case) or c) press submit on my apps the day the LSAT scores come out (depending on the score, of course)

    (My apps are all ready to go right now and I’m planning on applying to most T-14 schools)

    I apologize if this question is pretty redundant to my previous question. I guess I am just really anxious and appreciate the concrete responses you are able to provide me.

    Thank you.

  • Dave Killoran
    September 06, 2017 at 2:39am

    Hi AH,

    A broad overview of tutoring can be found in this video: https://vimeo.com/118956433. But, when I’m asked about tutoring by students, I usually look at what I know about the student–or ask additional questions–in order to pair students with the best tutor for their situation. With the live online platform we use, the beauty is that we can use the best tutors from around the country no matter wherever they can be, and the interface allows them to speak with you in real time and pull material from any of our many publications and courses.

    As far as the question types you mention, I have tons of tips for them, and they are in our books, our courses, and some are even on this blog (Use the Search bar at the top of the blog and do a search for certain words and you will see that we have articles on just about every topic under the sun!) and our forum (https://forum.powerscore.com/lsat/). But to try to post all the tips here would require hundreds of pages, and wouldn’t allow me to see what it is you *don’t* know, which are the tips you actually need to hear. Does that make sense?

    As far as submitting apps, until you have an LSAT score in the system, it doesn’t matter when you press the button–no law school will review your app until *every* single piece of your application is in place. So, waiting until the just a few days before the LSAT scores come out is just fine!

    No problem about the questions–I’m happy to help! It’s a confusing process and it’s often hard to tell what’s true and what’s not, or alternately, what the best pathway is out of the many that are possible 🙂 I hope this helped–please let me know if so!

  • Amy Gomez
    October 05, 2017 at 5:06am

    Hi Dave,

    My name is Amy. I took the LSAT in September I plan to take it again in December. I started the Power Score LSAT course in July. I finished the course with a 160. By August I was at 163. I hit the wall in mid-august at 165. I want to push for a 170(+) for my December test.

    I used a lot of my practice tests during my September preparation. I’ve been taking some of the older practice tests, some of which I’ve seen before. Though there may be one or two questions I recognize, for the most part, there is nothing that seems obvious to me while I test. Are these test still good indicators of my performance?

    LG is my weakest area. I usually get -5/4. I struggle with grouping games, in particular, both timing and conceptually. Grouping takes me the longest and is where I am most prone to unforced errors. Advanced linear games are a struggle regarding time as well. The other games are not particularly difficult for me, but I would like to become better with my timing. I often push the 9min mark on most games. As of now usually, I only have 6 min left for my final game. I’d like to fix that.

    My LR’s have been -4/-6. Any tips for pre-phrasing? I find it challenging in certain questions or when I am in a crunch for time. The question types that I commit most errors on are the must are must be true & most strongly supported, justify, parallel & parallel flaws, assumptions, and MOR. When I get these questions wrong I find that the correct answer was one I had as a strong contender. How do you suggest I deliberate between two answers in these cases? Lastly my powerscore instructor was adamant that the we narrow down our first 10 questions to 10 min. I find that I push the limits of those 10min. Any tips for speeding up? The frustrating bit is that sometimes I’ll take 12 min on the first 10 questions and get -2 on those 10, and I will spend the same amount of time on the next 10 and get them all right. I realize the first 10 are the easy ones I just don’t understand why I have such a slow start there.

    Lastly, my RC is my strongest section. Usually, I have -3/-4. I would like to narrow those errors slightly, maybe to -1 or -2. I struggle with humanities. I’ve started to drill those RC’s more, and I’ve seen some improvement. I seem to commit errors on viewpoint questions semi-regularly. Any tips? How do you suggest I improve my speed here as well? I always find myself at a bit of a rush towards my final passage questions. I find that I make more errors here because the time adds a slight panic to my thought process.

    Thanks in advance for your help!

    PS: I accedently posted this on

    Amy

  • Jenna Crescibene
    November 28, 2017 at 8:32pm

    Hello!

    My name is Jenna, I took a prep course over the summer with Test masters. My first time taking it dry was a 139, throughout the course I took multiple diagnostics and scored between 144-148 was my lowest and highest. I took the LSAT in September and received a 141. I was extremely disappointed with this outcome and did not expect such a low score. I am looking to attend law school in the upcoming fall 2018 so my next plan to retake the test in February but have exhausted a lot of my money towards the class and cannot really afford a high priced claasses/books. I was wondering what steps I need to take in order to study and do better on the February test.

    Thanks for your time and help

    Jenna

  • Dave Killoran
    November 29, 2017 at 5:47pm

    Hi Jenna,

    Thanks for the question! I’m so sorry to hear that things didn’t go well for you over at TestMasters. There’s a few things you can do:

    1. Use the full tests and LSAT questions they gave you to keep studying. Try to get access to every questions you can, and ultimately do as many as possible.

    2. Take advantage of the many free resources we have online. For instance, we offer the following to all students:

    Free LSAT Self Study Site. There are multiple study plans here, online test proctors, and access to our scoring analysis system to help you identify problems in the practice tests you took. The Study Plans would be of particular use to you since they are built on how much time is left until your test. They organize your studies in a specific way but also allow you flexibility.

    PowerScore LSAT Forum. When you have questions about LSAT problems or concepts, or how to apply a certain technique, come here. We’ve answered thousands of questions already, and daily we answer questions from our students (I spend a fair amount of time there because I like to see what students are asking about and to help when possible).

    PowerScore Free Help. On top of everything else, we offer a library of articles about the LSAT, including discussions of each section, specific methods, free webinars, and on and on. there’s one for admissions too.

    That’s just a start, but there are tons of resources that we offer at no cost to you that help you keep studying and learning.

    3. I know you took a course already and it didn’t help, so I won’t recommend another one (and obviously cost is a factor too). However, I still think it is worth suggesting that you consider our LSAT Bibles. Compared to the course you took they are inexpensive, and they are designed for students studying on their own, meaning that we explain each idea in step-by-step detail. But don’t take my word for it–read the comments of other students who used the books either here, or on Amazon. At least in my opinion, you need something that will help make these ideas clearer to you, otherwise you will be spending most of your time simply trying to create techniques for yourself. Think of it like this: if you had to learn calculus on your own, would you just look at calculus problems and try to reverse engineer them, or would you buy a book that would help explain how it all works?

    Hopefully this helps you get back in the groove. Good luck and please do not hesitate to let me know if you have any questions!

  • Alexandra
    May 29, 2018 at 8:37pm

    Hi Dave! I have a couple questions for you and thank you in advance for your help.

    First: I took the most recent February LSAT and still have yet to hear back from my top choice school; my application was fully ready for review March 7th. I’ve read a lot of articles that say I should not contact the law school unless they contact me first, but since it’ll be June in a couple days and other law schools’ seat deposits are due in less than two weeks, would it be okay for me to contact that school to check in? If so, what exactly should I say?
    Second: Is taking the LSAT three times too many? I took my first LSAT September 2017 and retook it February 2018. I managed to increase my score by 8 points, but I’m still not pleased. I’ve gotten into okay schools but none that I’m super excited about. When I was studying for the previous two LSATs I was drowning in responsibilities: working a demanding job 22 hours a week, was the President of an organization at my University, involved in three other organizations, while taking 17 credits. I honestly only studied 1-2 months for each LSAT and I know that’s my fault. I just graduated a couple weeks ago and am contemplating retaking the LSAT again this coming November. I would have zero distractions and a lot more free time for me to actually dedicate myself to getting the score I know I can get. I’m just scared that taking it three times will look bad to law schools.
    Third: My last semester of college I struggled really hard with a class and ended up getting a D which lowered my cumulative GPA down .05. Does having a D on an undergraduate transcript look horrible and if I were to take the LSAT again in November, will this D negatively affect my chances of getting into a top 30 law school?

    Thanks, again!

  • rebecca schultz
    May 29, 2018 at 9:20pm

    Hi Dave,
    I just wanted to check in and let you know that last year I took my LSAT and scored a 160 – not my personal best, but not bad either. It really took a lot of mental preparation on my hand – I have always struggled with intense exams, but I just worked on remaining confident and did NOT do any practice problems the day before the exam. With a 3.6 gpa in finance, I secured a full-tuition scholarship to a great school (one that recently joined top 50). I am very happy with that decision and wanted to let you and anyone else who is interested in scores versus scholarships know how my situation worked out. Also, a piece of encouragement to others: law school is really hard and rank is not absolutely everything. Even as a full-tuition merit scholar, I still struggle to perform average in my class. But, I still found summer work because I persevered and was honest about my interests. If you did not grow up in a law-school family, it will probably rock your world the first semester; it will take time to get used to this environment. Although I was accepted to other higher-ranked schools, I’m glad I chose the one I did. But above everything, know why you wanted to go to law school in the first place and let that motivate you to keep trying (whether it be LSATs or exams). I hope everything works out for you guys! Good luck! And thank you Dave, for the mental preparation tips!

  • Dave Killoran
    May 30, 2018 at 5:37pm

    Hi Alexandra,

    Thanks for the message! Here are some thoughts:

    1. It will be okay once you hit June 1st to contact them. Simply explain the situation; they know other deposits are looming, so they are likely to make a decision soon anyway 🙂

    2. 3 times is *not* too many, and it’s not even a debate anymore. 5 isn’t too many these days. All that matters is the high score, nothing else.

    3. Ds aren’t great no matter when they occur, and you will likely have to write an addendum explaining what happened. In the event it was fatigue, taking a year off might work in your favor, to be honest. Otherwise, you are going form school to school, and a school would think there’s no reason to suspect your fatigue would disappear.

    Thanks!

  • Dave Killoran
    May 30, 2018 at 5:46pm

    Hi Rebecca,

    So nice to hear back from you, and congrats on not only getting past the LSAT but through your first year of law school as well (with a full ride)!

    You make a lot of great points here, and I’m glad we were able to help you in any way. Keep working hard and take some time to enjoy your success!!

    Thanks again for the update!

  • Ken
    June 24, 2018 at 10:33pm

    Hi Dave,

    I plan on taking the LSAT on the July 23rd date. I have been studying since early May using the Powerscore Bibles and Powerscore training books. I have been studying on average 30 hours a week. My diagnostic was a 145. I improved to a 150, but plateaued there. A few weeks ago I purchased some tutoring hours to help with the logic games section. Now I feel comfortable with this section and I’m now at a plateau of 156. In total I’ve taken 12 practice tests. I have been reviewing the logical reasoning sections by going through all of the questions I get wrong and using the tracker sheets. I was seeing patterns early on and I would review the concepts and practice them. Right now I can’t seem to find any patterns, or obvious area of weakness in any of the sections. The only thing I’ve noticed is a correlation between the likelihood of getting a question wrong and how difficult the question is (according to the % that get it right). This is regardless to question type and section. So all that really tells me is that I need to get more of the difficult questions right. That’s not exactly helpful, or an easy thing to achieve. I would like to get as close to 160 as I can, but I would be happy with 158. I would also like to note that I am approved for extended time (53 mins per section). I usually have a few minutes to spare on the logical reasoning sections; however, I find that I’m usually rushing on the last logic game and the last RC passage.

    So given this information, what do you suggest I do for the next four weeks until the July LSAT?

    Thanks,
    Ken

  • Dave Killoran
    June 25, 2018 at 9:03pm

    Hi Ken,

    Thanks for the question! It’s difficult for me to give an exact assessment of what you should do since there are still a lot of unknown variables, but one thing that jumped out at me was that you’ve taken 12 practice tests. I’d suggest doing more PTs, for several reasons:

    1. Getting battle-ready under timed conditions will make you more prepared and comfortable during test day.

    2. Doing more PTs will expose you to more questions, including many high-level difficulty ones.

    3. The review you do (and I suggest Blind Review) will help you find more of the patterns and problems that are troubling you. You know there were patterns before, and there will still be patterns in the harder questions as well, it just might take a bit to spot them.

    So, my advice would be to ramp up the timed PTs with blind Review over the next few weeks, and max out on both the number of questions you see, and the time spend getting deeply into those questions. There are some thoughts in here that might help as well.

    Please let me know if that helps. Thanks!

  • Tayler Berlin
    September 08, 2018 at 1:36am

    Hi Dave,

    I know this thread is a little dated but I’m hoping I can still get some feedback.

    I started studying for my LSAT at the beginning of July by taking the Powerscore live online course. My diagnostic was a 153. I took another practice test midway through the course and had jumped to a 161. However, towards the end of the course, I tested back at a 153. I was planning to take the September LSAT, however I was so disheartened by my decline that I pushed my date back to November.

    Prior to pushing my test date back, I was doing around 10 hours of studying each week. However, once I tested back at a 153 and pushed my test back, I developed a bit of test anxiety and have since been taking a “break” from my studying.

    I am ready now to get back into the swing of things so I will be more prepared for my upcoming November test. Since I have already completed the Powerscore course, what do you suggest as a study schedule for the next two months? My goal is to score a 170+. I know I can get there, I just need help identifying how to best spend my time. My initial thought is to focus on practice tests, maybe 2-3 a week? What are your thoughts on this?

    – Tayler Berlin

  • Dave Killoran
    September 11, 2018 at 10:49pm

    Hi Tayler,

    Thanks for the message! A few thoughts for you:

    I wish I’d had a chance to talk to you when you scored that second 153. that kind of “reversal of fortune” happens frequently, and it shouldn’t throw you off too much. I wrote a lengthy response to a similar question over here. Take a look at that and read some of the articles I link in there–I think they will help you!

    Ok, moving on to your schedule, here’s what I want you to read first. That outlines the framework for preparing for a retake (which is kind of similar to what you are doing at this point, so it will match what you need perfectly). Your initial inclination is a good one: focus on practice tests and especially focus on reviewing those exams and linking concepts together. The article explains it all 🙂

    Please let me know if that helps. Thanks!

Comments

  1. Ken says

    March 26, 2023 at 5:30 pm

    Hi all,

    I know this is an old thread but I’m hoping for some advice! I am taking the June LSAT (I’m open to pushing it back to August though I do not want to unless I have to) and I began studying in mid-January. I am using the PowerScore on-demand course and just finished Lesson 5. I usually study around 10 hours per week. My diagnostic exam was timed, but my PTs have been untimed for right now. I also own the PowerScore bibles (2020 versions), but have not been using those for my studying right now; if those would be helpful for me to use I have them on hand.

    My diagnostic was a 161 (RC: 20/27, LR 20/25, LG 17/23).
    I took my first PT about a month into studying and scored a 156 (RC: 20/27, LR 13/25, LG 17/23).
    I just took my second PT today and scored a 160 (RC: 19/27, LR: 16/26, LG: 20/23).

    My target score is 170. I am looking for advice on how to go forward from here, especially in my LR section. While studying between my first and second PTs I focused heavily on drilling LR questions and on my second PT I only missed one of the easier questions. The remaining questions that I got wrong were all 50% or less on the difficulty percentage. The LR question types I am missing seem to be a mix of all kinds and almost always in the second half of the section. Should I be targeting the more difficult LR questions now and are there certain skills I should be focusing on for these challenging questions? I am also wondering if these increments are where my skills should be at given where I am in my studying, course, and overall target score.

    Thank you very much in advance!

    Reply
    • Dave Killoran says

      March 27, 2023 at 7:21 pm

      Hi Ken,

      Thanks for the message! There’s actually a lot to like in your position currently, and room for growth.

      When I look at your LR score, one thing I wonder about is your RC score. So much of LR is based on reading and comprehending, and you are good in RC but not yet elite. Is your weakness there based on reading speed? Or tough passages? Or something else? Understanding that would help me better frame what’s occurring in LR so please let me know when you have a chance.

      Thanks!

      Reply
      • Ken says

        March 27, 2023 at 10:07 pm

        Hi Dave,

        Thank you for your response! I’m missing a similar amount of questions across each of the passages, and the missed question types are a bit varied though I did notice I am missing a lot of Global Reference Must Be True questions. Let me know if there’s anything else I could be looking at in my results!

        I do feel like I grasp an understanding of the passages as I read, but I also have not worked a lot on incorporating the reading strategies from the Lessons yet. I also have not focused on drilling any RC outside of the Lesson homework (which I think can be attributed to my familiarity with the Reading Comp format compared to LG and LR, so I’ve focused more on practicing in those areas first).

        Thanks again for your help!

        Reply
        • Dave Killoran says

          March 28, 2023 at 1:38 pm

          Ok, thanks! The reason I ask is that one analysis we make is to look at how your general reading ability is affecting your LR performance. For example, lets say that we are looking at a student result and find the following:

          * You have very good RC and not so great LR: that suggests you are a good reader who simply doesn’t understand the principles of LR, which you can then learn. Lots of upside here since you can spend a lot of time learning how argumentation works.
          * You have very poor RC but reasonably good LR: here you are outperforming your reading ability, which suggests you know the concepts really well; spend less time there and more time on fundamental reading and comprehension principles.

          In your particular case, the numbers are fairly similar, so I was trying to get more info on what’s happening underneath in RC to see if that was suggestive of a particular course of action. What you are telling me is that it’s probably a combination of both: you need to focus a bit more on RC because your grasp of the details–the facts of the passage or argument–aren’t quite at the level you want for the LSAT, and there there are likely some areas inside LR argumentation that you still need to lock down. That’s a less clear pathway, but it’s not uncommon–its the sign that you are still working along the steep learning curve of this test!

          Fwiw, the “variety pack” of LR questions you are missing also suggests this isn’t tied to a specific issue but a more general “reading for detail” related issue. When difficulty rises later in a section, you are missing some of the little pieces, and that’s causing you to miss the problems most likely.

          How does that sound to you as far as your situation? Thanks!

          Reply
          • Ken says

            March 28, 2023 at 9:38 pm

            Yes, I realized this is definitely the issue I am having! I just finished reviewing my last PT and there were so many LR and RC questions where I missed little details, but once I noticed them the correct answers made sense. I can see now that reading for detail is a common thread in a lot of my errors. There were also a couple questions I missed in LR where I didn’t see a specific word in a stimulus or certain indicators that would have changed my final answer choice, so that is something I can pay closer attention to on my next PTs. Thank you for your help with this!

          • Dave Killoran says

            March 29, 2023 at 5:59 pm

            Anytime, glad I could assist! Keep working hard 🙂

          • Ken says

            April 16, 2023 at 7:33 pm

            Hi Dave,

            Hope you are well! Apologies for the back and forth on this, but I did have a follow-up question for you based on our recent discussion. I really focused on reading for detail while studying the last few weeks. 2 weeks ago I took a PT and scored a 161. I worked through more of the PowerScore on-demand course (just completed Lesson 7) and today I took another PT and scored a 167! I scored -1 LG, -6 LR, and -3 RC!! I am so happy with my RC score in particular, especially because it was also my Experimental section and I only missed 2 in that section as well!!

            With that being said, do you think it could be a good idea for me to push my June test to August? My target score is a 170, but I’m confident I can hit higher than that with more practice. For context, I am looking to apply to the majority of the T14, so scoring higher than 170 would be ideal. I should be finished with the course videos by the end of April, which leaves all of May for drilling and review. I have a full-time job, though, and I am worried I may not be giving myself substantial time for PTs and concept review (with my schedule, I only have time for PTs on weekends). I also need to incorporate time. I have been getting quicker, but have held off on taking timed exams until I felt comfortable with most of the concepts. I imagine my score will take a bit of a dip once I start timing myself (which I plan to start doing with my next PT). In summary, I want to give myself the best chance at maximizing my score and was wondering whether giving myself May-July to review and drill would help to do that. (I am conscious of burnout, but I think May-July would be less intense than the studying per day I am currently doing).

            Thanks so much in advance!

          • Dave Killoran says

            April 22, 2023 at 1:49 pm

            Hi Ken!

            Congrats on the progress–that is great to hear 🙂 What I’d say is to wait to make that decision until sometimes in later May. At that point, if you feel like there are more points to be had by waiting, then push to August. August is plenty early for apps to be put in, so if there’s any advantage to you personally by waiting, you should have no hesitation in doing so!

  2. Kaitlin Pogue says

    January 15, 2023 at 3:08 pm

    I know this is old, but I wanted to post here and ask advice anyway!

    I took my first diagnostic in November but didn’t really start studying until right before Christmas (Dec 22nd). I used the Kaplan On Demand course and took PrepTests.

    I plan to take the June LSAT. I just took the January one yesterday, and I didn’t feel BAD about it, but I haven’t even reached my goal score in PTs, much less on the real deal. My goal is to get into the 170s. (I’d be happy with anything in the 170s, but the higher the better. My goal is a scholarship plus a stipend at a modest school rather than paying a lot at a more prestigious school.)

    Scoring History: (all LRs with an asterisk include two sections worth, not just one)
    November 18 PT 81 (Diagnostic): 153 (-12 LG, -20 LR*, -7 RC)
    December 24 PT 89: 156 (-12 LG, -15 LR*, -6 RC)
    December 26 PT 88: 158 (-11 LG, -9 LR*, -11 RC)
    December 26 PT 77: 163 (-5 LG, -12 LR*, -5 RC)
    December 27 PT 78: 159 (-8 LG, -15 LR*, -7 RC)
    December 30 PT 68: 159 (-7 LG, -10 LR*, -12 RC)
    December 30 PT 70: 157 (-7 LG, -17 LR*, -8 RC)
    January 1 PT 45: 165 (-3 LG, -11 LR*, -6 RC)
    January 2 PT 76: 162 (-2 LG, -14 LR*, -8 RC)
    January 8 PT 85: 159 (-4 LG, -12 LR*, -11 RC)
    January 9 PT 86: 166 (-2 LG, -9 LR*, -7 RC)
    January 13 PT 93: 162 (-4 LG, -5 LR, -8 RC)

    I have the Powerscore Bibles and Workbooks all coming in the mail, I have the Logical Reasoning Loophole, the LSAT Trainer is coming in the mail, the Logical Reasoning book from LSAT Dragon is in the mail, I have Kaplan Unlocked, and I have a subscription to 7Sage as well as one to Kaplan that expires in March. I will be getting your Testing and Analytics monthly subscription too so that I can complete the drills that go along with the books in the 4 month study plan.

    Any advice you have for me is very welcome, especially in Reading Comprehension as I’ve had such a dip and so much inconsistency! I feel I’ve improved quite a bit in LG and LR, but RC has stayed the same or been worse, not ever really any marked improvement on it from my diagnostic.

    I appreciate you taking the time to read all this!

    Reply
    • Dave Killoran says

      February 8, 2023 at 2:25 pm

      Hi Kaitlin,

      Sorry, I just saw this post but here are some additional thoughts.

      First, you have generally improved, so that is good news 🙂 Your LG is definitely better. Your LR and RC still need work though, as I’m sure you noticed. One thing I will say is that your pace of testing is a bit too compressed for my liking. From Dec 24-Jan2, you took 8 tests, and it’s so hard to properly review a test in even 2 days, and you were trying to take and review one per day for the most part. Space your testing out a bit more so you have time to properly review every question. That will actually improve your score more than just taking test after test 🙂

      Second, one thing I’m seeing is that you are getting materials from all quarters, and that can be confusing for students. Some methods conflict, whereas others agree. Sorting that out can actually be time-consuming and problematic. For example, PowerScore and Loophole largely agree (I edited Loophole and know Ellen well) but Kaplan uses a different, and in my opinion inferior, approach. I’d drop Kaplan, Trainer et al to the bottom of your study list.

      Last, LR and RC have some connections, but the way in which you read each needs to be different. LR is so detailed whereas RC is often about the big picture and knowing the structure. Focus first on LR so that you know you are efficiently reading for detail; that will then help feed into RC since you’ll begin naturally picking up more detail even though you are actually looking at bigger picture items.

      Thanks!

      Reply
  3. Mary says

    October 27, 2020 at 10:17 pm

    I have less than 10 days until the November exam, and I am freaking out – aka the last thing I should be doing right now.

    I’ve been following the 6-week study plan – I took the August exam and completely bombed it after 5 months of studying with all the Powerscore books (this was my fault, I had a huge anxiety attack in the middle of the exam). After 11 practice exams, my average is 158. My LG score is usually around -2 or -3, LR is anywhere around -6 to -9, then RC is brutal – just started getting it to -10. I’ve been using your online portal and notice I struggle with CR, MBT the most on RC.

    I am in desperate need of at least a 160, and a 165 would be amazing.

    Should I continue on the 6-week study plan, and take 4 more exams before the real deal? Or should I do drills? And what kind of drills should I be doing? Before August I was doing all kinds of drills and I think I burned myself out way too much before it.

    Ahh! Thanks so much in advance.

    Reply
    • Dave Killoran says

      October 30, 2020 at 1:39 pm

      Hi Mary,

      Thanks for the message! Let’s start by taking a deep breath and focusing on the idea that no matter what happens in November, you will have options still 🙂

      Let me say that as far as November, and seeing how burnout played such a negative role before, that I would NOT take 4 exams between now and then. Instead, I’d do something counterintuitive and slow down. That will keep you fresher and reduce the pressure you feel! Focus on those areas you know are the biggest area of concern, which is going to be mostly RC and then some LR (which is nice since working on one section can help the other a bit). Don’t worry about drills–just do questions and focus on really understanding each one that troubles you.

      Hopefully, given how you feel, the above helps relax you a bit more, which will give you the best shot to break that 160 line!

      Reply
  4. Mikaela Ackermann says

    October 9, 2020 at 5:05 pm

    Hi Dave!
    I have been going through the forums and articles–I appreciate all the wisdom and advice you offer through the site!

    I am registered to take the November 2020 Flex, so I am onto my last month of studying. The nerves are catching up to me, especially because I feel worried that I will not reach my goal. My target score is a 168+.

    I began studying June 1, 2020. My cold diagnostic was a 146. I purchased the 3 bibles (very helpful), and saw my scores increase immediately. I am studying usually 2 hours a day, 5-6 days a week. By mid July, I got my first 160. I had a few more high 150s, but by mid-August I began consistently scoring 160-163.

    However, this began my month long plateau, wherein I scored 161 almost every exam I took. This was very frustrating, so I took a step back to drill and revisit the ideas in the bibles. This seemed to help, because I recently reached 166 on three PTs in a row, which is the closest I’ve ever gotten to my goal. However, the last 2 PTs I have taken have all been back in the 161 range, which is discouraging and a step back instead of forward. I blind review every one of my exams, and the mistakes I made were silly and shouldn’t have tripped me up. My biggest issues are LR and LG. LR I get anywhere from 2-7 wrong, which is a huge range, and can be challenging to pin down. LG it’s usually around 5 wrong.

    For my last month, I want to channel the same increase that I saw when I made the jump from 161-166. I really think I can achieve my goal, I’m just feeling stuck on how to do it! Do you have any advice for the final month of studying that can get my that last jump? And, do you think a month is enough time to get to my target?

    Thank you so much for all of your help, I really appreciate it !
    Mikaela

    Reply
    • Dave Killoran says

      October 9, 2020 at 8:00 pm

      Hi Mikaela,

      Thanks for the message! I have a few thoughts for you that might help:

      • A month can definitely work here for you. It will depend on some things, but you’ve already posted multiple 166s, so you aren’t far from your target range.

      • Despite time being relatively short, you might need a break here. This sounds a bit like burnout, which cycles into anxiety, which leads to more burnout. That could explain the silly mistakes. Maybe 4-5 days completely off from the LSAT would help? It has for others.

      • For some ideas on how to bounce back form those lower scores, perhaps listen to this podcast: LSAT Podcast Episode 63: Reversal of Fortune: How to Recover From a Bad PT.

      • And to focus on some of the finer points of improving your score, this article–focused on retakers but still useful to anyone–might help: Retaking the LSAT? What to Do Differently to Raise Your Score

      The above may give you some thoughts, but it could also be that it will be hard to determine exactly what’s causing you to stumble. That may be the time to connect with a tutor who can analyze what you are doing and help push you over the top 🙂

      I hope that helps. thanks!

      Reply
  5. Astha Jindal says

    September 5, 2020 at 10:20 am

    Hi. I am Astha, I am planning to give the Int’l October LSAT. I first decided to prepare for the LSAT in April, but didn’t study much because I wasn’t sure about my decision. But nevertheless, I built up my base knowledge and gave my July LSAT-flex without studying at all the 2-3 weeks before. I got a 147. (Which I was satisfied considering I didn’t study at all).
    Later, one I was sure, I purchased the Powerscore LR, as LR was my weakest section (Other books weren’t available on amazon in India). But realised self study was not working for me and thus took further action in August.
    I have registered for the October LSAT and I wish to score around 168-170 as I have a GPA of 3.3-3.7 (65% from Indian University). My diagnostic in August was 145 (BR 156). I have also taken PT 45 (143, br- 158) and PT 46 (149, br- 159).
    All of my fellow LSAT takers are advising me to stop trying to get 168-170 as it is impossible to have this kind of jump in 30 days. I am currently studying 8-10 hours a day (60 hrs approx. a week), and I willing to put more hard work. I am very discouraged, please give any advice as to what I can do to reach my goal score. Or if it is even possible.

    Problem Areas: I understand LG questions well, but face time constraints and I mostly rush through the last game. With LR I have a problem with diagramming conditionality, I understand validity but find it hard to diagram them which effects my MBT, Sufficient assumption questions.
    Major problem is RC, I miss around 15-19 questions.

    Reply
    • Dave Killoran says

      September 6, 2020 at 7:46 pm

      Hi Astha,

      Thanks for the message. My best advice here: ignore what everyone else is telling you. they don’t know you and they can’t possibly have any insight into what any one person is capable of. In fact, please go read this article right now: https://blog.powerscore.com/lsat/bid-326433-tom-brady-and-the-lsat/. After that, go read the Forum thread I had with a student in a very similar position to yours: https://forum.powerscore.com/lsat/viewtopic.php?f=2&t=4666.

      The above aside, the jump you want in 30 days is a big ask. Your blind review scores aren’t really applicable here since the test is under time and you don’t get a review, so what you are really wanting is 149 to 170 jump in a month. The LSAT is a tough test to cram for (meaning, it’s very hard to do so), and problems with RC and conditionality are hard to fix within a short time-frame. I still think you shoot for 170, but realize that for the vast majority of people, they’d need more time. I’d really dive into your analytics in the next month and focus on reading–that’s a clear hindrance for you! Also, read this article on LR diagramming, because most people do too much of it: https://blog.powerscore.com/lsat/bid-333431-lsat-conditional-reasoning-when-to-diagram/.

      Thanks!

      Reply
  6. Shelly says

    September 2, 2020 at 10:57 pm

    Hi Dave!

    First off, I LOVE your podcasts!!! Thank you so much for providing another free resource, I’ve been listening to them every time I get into the car.

    I just wrote the August LSAT-Flex and know I did not do well on it, so I’m planning to write the November one, I’m just not sure how to proceed. Prior to my LSAT, I was scoring high 150s and got my first 160 on the last preptest I wrote before my LSAT!

    Test Date: November 7, 2020
    Study History: 4+ months
    Scoring History: diagnostic was 148, scored low 150s for months, then in my last month I finally started getting high 150s
    Target Score: Although it may be completely unrealistic, I would LOVE to get a 168-170
    Prep Material: Read all the bibles, and just ordered the workbooks as I did not have them while prepping for my August LSAT
    Problem Areas: I feel like everything…
    LG: can’t finish all 4 games, usually finish 3 and am able to start the fourth one (usually score 17-19/23)
    LR: Flaw, Must, Strengthen (usually score 15-19/25-26)
    RC: I feel RC may be my worst section… I find the passages are so dense and sometimes so complex that I struggle to sort through all the information (usually score 15-18/27)

    Reply
    • Dave Killoran says

      September 6, 2020 at 7:30 pm

      Hi Shelly,

      Thanks for the message and for the kind words! First, while I know it feels like everything is a problem area, it’s not. To get the scores you were getting before August says that in general you are doing really well in some areas! So, take some comfort and confidence from that 🙂

      Second, let’s start at LR. When you say Must, Flaw, and Strengthen are areas of concern, can you tell me more specifically how you perform on those. Do you get half right? Miss most? Occasionally have a bad section with each? Or just don’t feel great about them? IO ask because those are three very different types of questions, and knowing more will shed some light for me on possible systemic problems (as opposed to straight Q Type problems).

      Thanks!

      Reply
  7. Nikki A says

    July 28, 2020 at 2:49 pm

    Hi Dave,

    Just wanted to add one more thing.

    I don’t have any more study material. I’ve taken all the tests so thats another problem that I am dealing with. I don’t know if the scores I will get on practice exams will accurately reflect what I can get on test day anymore.

    Thanks!

    Nikki

    Reply
  8. Nikki A says

    July 26, 2020 at 6:16 pm

    Hi! I hope all is well and you are staying safe! 🙂

    I have completed two LSAT tests so far: June and July.

    I self-studied for both exams. I bought the test books online and practiced on those. I also listened to two Podcasts: Thinking LSAT and the Powerscore one. I took my diagnostic in December 2019 and scored a 148.

    I started studying in January, but it wasn’t until around March when I realllyy started putting in work. I would study at my desk for about 10 hours on weekdays and on weekends I would put in at least 3-5 hours per day. When I wasn’t at my desk, I would watch LSAT videos on YouTube and listen to more LSAT Podcasts. I took a full practice exam at least twice a week. I found that my score was increasing as I was scoring in the 150s and eventually cracked a 160. There came a point where I was consistently scoring a 164 on my practice exams. But then I started to hit a wall and my score started to decrease. I did research and found that I was getting burned out so I decided to take a step back from studying. After a couple of days off, I took another exam and my score picked up again.

    The last few weeks leading up to the June exam I was still scoring a 164 consistently. However, when I took the June exam I ended up getting a 159. I remember the hardest section being the Reading Comp, so I decided to mainly focus on that for the July exam.

    Leading up to the July test I was scoring my personal best on PTs. I took two tests the week of the exam and got a 168 and a 170 (granted, I had taken the logic games sections before so I didn’t really take those scores too seriously). I decided to stop taking practice exams after that 170 and just moved to individual sections. On July 12 I took the exam and had my first anxiety attack in the middle of the logical reasoning section. I don’t really know what happened. I wanted to ask the proctor if I could get my inhaler but I was worried he wouldn’t pause my test so I just dealt with it. I finished the test and talked to people around me who said that I had basically hit my potential at a 159 and that I shouldn’t study anymore because my score wouldn’t increase.

    My dream score is a mid-high 160s.

    I have registered for the August exam and I think I should take it. I am just worried that law school admission officers wouldn’t want me because with August I would have taken it three times. Also, I’m worried that I might have done worse on my second test than my first and I’m sure that isn’t looked at too positively by admission officers.

    My typical score on PTs would be:

    LG: ranges from -0 to -3 wrong
    RC: this ranges a lot for some reason but I could do as well as -4 to -7
    Timing is a big issue here. I found that if I take notes while I am reading it helps me understand the material
    better though, but then I worry thats why I am wasting so much time.
    LR: usually get -5 to -6 wrong here

    Anyways, sorry this was so long haha. Thanks so much for your help!! 🙂

    Nikki

    Reply
    • Dave Killoran says

      July 26, 2020 at 8:03 pm

      Hi Nikki,

      Thanks for the message! I have a few thoughts/questions here:

      • You mention having “the test books” but that doesn’t really give me a good sense of what you have been studying. Are those our books? Someone else’s? Just tests?

      • One point of concern I see is that you are using so many different resources that I’m not sure how cohesive that is. Some resources out there work very well together, others less so. Your regimen sounds like a lot of LSAT time, but I can’t see whether that would all add up in the optimal way.

      • I’ve often said that the opinions of other people about YOUR potential are irrelevant. they aren’t you. So I’m not sure who told you that your ceiling was a 159, but I’m skeptical of them base don what you’ve told me. Don’t listen to the opinions of others o things like this. This blog might help more of my thoughts: Tom Brady and the LSAT

      • 3 takes is nothing these days, so don’t worry about that at all! A higher score would even outweigh 6 takes, for example, so 3 is not notable. And a second lower score is also relevant: all that matters is your highest score!

      I hope the above helps. Thanks!

      Reply
      • Nikki A says

        July 28, 2020 at 2:45 pm

        Hi Dave,

        Sorry I should have been more specific. The “test books” that I was referring to where the “10 Actual, Official LSAT PrepTest” books.

        Thanks so much for your help!

        Nikki

        Reply
        • Dave Killoran says

          July 28, 2020 at 5:19 pm

          Hi Nikki,

          So, it seems you are doing well with studying just tests, but it looks like you haven’t really delved into any prep materials (sorry, podcasts and videos don’t count–they only hit pieces of any approach, and don’t show everything we teach in cohesive fashion). I’d recommend that to improve, you go out and obtain some actual study guides like the LSAT Bibles. They help teach you a comprehensive approach to this test, and they will help increase your exam knowledge and stabilize and raise your score when used properly.

          As for additional materials, aside from the fact that the Bibles and Workbooks contain numerous drills, it’s likely you can get more from the tests you have seen. This article expalisn how that works: https://blog.powerscore.com/lsat/bid-239160-retaking-the-lsat-when-you-ve-seen-all-of-the-practice-questions/

          I hope this helps. Thanks!

          Reply
  9. Denise Salazar says

    July 15, 2020 at 11:56 am

    Hello! I hope you all are still available to help!

    My test date is August 29 – flex mode.

    I did self study before my test in September 2019 and my mind was nit in the right place. I was unhappy with my score of 145 so I decided to take a course. In November 2019 I started a blueprint course that was two months long.

    Scoring History for 2020 :
    April 26 – 156 – PT 56
    May 4 – 152 – PT 67
    May 19 – 154 – PT 57
    May 25 – 150 – PT58
    June 1 – 157 – Khan Academy Diagnostic
    June 8 – 154 – Khan Academy
    June 15 – 151 – Khan Academy
    July 11 – 153 – PT 59

    My goal score is a 161-163

    The order of the resources I have used are as follows:
    summer 2019 – Sept LSAT Self study with Powerscore bibles and 6 month stchedule:
    https://lawschooli.com/shop/lsat-study-schedules/6-month/
    November 2019 – January 2020 Blueprint
    I studied after January but not consistently and used Blueprint for another month, then I started using my PT books and targeted certain question types I needed help with. The way I would find these questions was through the 6 month study schedule.
    From April 26 – May 30 I went harder and was studying maybe 2 hours a day with the 6 month study schedule. I felt like my scores were inconsistent and I needed to switch it up and started Khan Academy from May 30 – Now. I just purchased the digital exams from Powerscore and I love it so far!

    I usually score :
    17-21 on LG
    10-19 on LR
    10-15 on RC

    I am now studying 7 hours a day in 2hour intervals. It actually does not make me feel as drained as I thought. I usually focus every 2 hours on a different part of the test. LG is always about speed or ordering/grouping slowing me down. LR Is always Flaw, Deductions, PI, Weaken, “Role” or “Describe”, and Assumption Question types. RC is just so hard for me to keep up. I get to 3 passages and I miss so many. I have already established that I will be using tips on your site such as writing summary and predicting questions and mind mapping. I am sticking to studying from Khan Academy because I am enjoying it. All of my testing regarding sections or full tests will be on Powerscore.

    I feel as though every time I get a score in the high 150’s range, it is pure luck. I feel like I am hitting a ceiling and I know how to get through, but every time I feel like I have made progress and take a test, I find myself amused at how little (or none) I improve.

    Thank you for all the great resources and help! Can’t wait to hear back!

    Denise

    Reply
    • Denise Salazar says

      July 15, 2020 at 6:36 pm

      Hi Dave,

      I wanted make an edit, I just went over many articles on your site. I am adopting Marvin’s schedule. It seems very focused on getting everything covered while studying. The only worry I have is that, because I still struggle on specific types of LR questions, should I incorporate drill sets of my “Hard” question types? Or do you think it is fine if I just take a full LR timed section then make sure I grasp why I got it wrong and why the correct answer is correct? Is there a more recommended approach?

      Also, how many PT should I take a week?

      Thank you so much again.

      Denise

      Reply
      • Dave Killoran says

        July 18, 2020 at 7:48 pm

        Hi Denise,

        Thanks for the message! Good, I’m glad you found Marvin’s posts–I would have suggested those to you as a nice helping outline to what we offer 🙂

        I think what you can do is doe the sections, but then pay special attention to the higher difficulty ones, since those are indicated within that stats.

        I suggest 1, 2, or 3 PTs a week, whichever works best for your schedule. Just make sure you can adequately review each test–that is actually the controlling factor in how many you do 🙂

        Thanks and good luck!

        Reply
  10. Ricardo says

    July 1, 2020 at 12:28 pm

    Hello,
    I am registered to take the LSAT flex July 12th and i am aiming to score in the 150s. This is my 4th time taking the exam. First time, I ever took the LSAT I scored a 139. 2nd and 3rd time I scored a 141. I took a break from the LSAT for a while due to family emergencies and work load from the law firm I work as a Legal Assistant. I’ve been studying two hours a day for the last two months using all the Powerscore bibles and 2 month Study Plan; I also have the workbooks along with bibles. I’ve also been Utilizing the Digital exams on your website to get the analytics of where I am weak. The last 4 exams i boosted up to a 147-148. The last practice exam I took this past weekend, dropped to a 145. I think I was putting to much pressure on myself to break 150 and also I was exhausted from work. I have two weeks left and I refuse to not get a 150. This test has been a pain in my ***. Any advice to finally break through to 150? Any advice will be greatly appreciated.

    Reply
  11. Maria K says

    September 6, 2019 at 10:49 am

    Hello, Dave

    My name is Maria. I am registered for the September, October and November 2019 LSATs. I started studying on June 1. My diagnostic was 145. I took a Blueprint class and also got all the Bibles. Right now I do about 50 hours of studying a week and I am averaging 161. My goal score is 173+ (Ideally 180) because I want to get into one of the top 14 schools (preferably HLS). I have taken about 30 PTs. I make sure to blind review them right after I finish.
    I’m going to try to summarize the scores since I have taken so many.
    145, 150, 151, 153, 159, 150, 157, 161, 163, 159, 159, 162, 161, 161.
    I have all the tests from 1 to 87 including A, B and C. I take about 4 a week (again, blind reviewing them all). I don’t feel burned out at all, and I actually enjoy studying and look forward to taking tests. It can be a little frustrating when I get the same score 3-4 times in a row but I’m always happy to learn from my mistakes. I’m averaging -6 in RC. -7 in each LR (my best has been -4) and -4 in LG. So, do you think it is reasonable to expect anything close to my goal score by September? I registered for November just in case, but I’d love to hit my goal by October at least. Do you think that sounds possible?

    Thank you so much for your help!

    Reply
    • Maria K says

      September 6, 2019 at 10:56 am

      I forgot to mention. I have only been studying that much for a month. In June I was averaging 31 hours a week, in July I barely studied because I was taking a summer class. In August I started doing about 50h a week. At the beginning of August I was at a 154. Thanks!

      -Maria

      Reply
    • Dave Killoran says

      September 6, 2019 at 5:00 pm

      Hi Maria,

      Thanks for the message! I’m impressed with your dedication, and always feel that hard work is required to reach any high LSAC score. So, that’s a great sign 🙂

      As far as improving, right now you seem to have settled into a comfort zone of Blind Reviewing. That’s great, but by itself it simply tells you why you missed problems, it doesn’t necessarily lead you to methodological or conceptual deficiencies. So, make sure you keep studying the methods—most students never reach full comfort with them, and by full comfort I mean being able to explain them to others clearly and without delay. I describe that approach in detail in Step 6 in this article: https://blog.powerscore.com/lsat/retaking-the-lsat-what-to-do-differently-to-raise-your-score/. That whole article would be of value to you as well, just ignore the part about retaking and use it for improving your score from where you are now.

      So, you’ve made great progress already, but keep mixing up your studying and don’t make it all BR.

      You can keep rising, jsut put in the time and focus on connecting the logical dots in these questions. Good luck!

      Reply
  12. Cameron McCrary says

    April 21, 2019 at 5:31 am

    Hi Dave!

    I hope you’re well!

    I am registered for the July 15, 2019 administration.

    I took my very first diagnostic test through a Princeton Review testing center back in the Fall of 2017 and scored a 138. I panicked and dropped all consideration of preparation until later in my college career (I was a Sophomore at the time).

    I decided to begin preparing back in January of 2019 and over the course of the month, I went through the concepts in the LG and LR bibles and finished about a third of the RC.

    Here are my practice dates and scores:
    February 2 | 152
    February 8 | 153
    February 15 | 155
    February 23 | 150
    March 3 | 155
    April 2 | 154

    I have been consistently scoring the following ways on each section:
    LG -10,-11
    LR -5,-6 (occasionally, there are LR sections that I will miss 13 or 14 out of the blue)
    RC -10,-11

    Aside from practice tests, I haven’t had much of an opportunity to study during the semester because of work and school. With the time I do have, I try to go over and re-work what I missed and explain LR questions. However, mid-May I should be able to dedicate much more time as I will only be working part time and I plan to follow the 2 month study plan.

    Would you have any recommendations for anyone hoping to score in the 167-169 range by July?

    Thanks,

    Cameron

    Reply
    • Dave Killoran says

      April 23, 2019 at 3:34 pm

      Hi Cameron,

      Thanks for the question! I have a few thoughts that might help:

      First, with your scoring, it’s sitting so consistently in that low 150s range that it suggests to me the issue is less about explaining or understanding questions, and more about not being where you need to be with the concepts and techniques. So, for something like LR, those sudden sections where you miss 13 or 14 indicate that they really tested certain ideas and you were shaky with those and paid a price. If I were to sit down with you and ask you to explain certain concepts, such as how Grouping/Linear games work conceptually and what’s important in them, or why Assumption Negation works, or how a Justify question is different from an Assumption question, etc, would you be comfortable explaining those ideas? I suspect not, which means that’s what you have to work on first.

      Second, inside your performance here, what are your strengths and weaknesses. Meaning, when I see you are -5 or -6 in LR, which question types are getting you most frequently? I ask because those are the ones you need to drill the most with, and to do repeatedly. Examine answer choices and look for similarities in problems. In other words, worry less about completing problems and worry more about understanding them fully. For example, you should redo questions you miss over and over until you know them in your sleep (this is especially so for LG, where redoing tough games several times is critically important!). If you aren’t certain which types you are weakest in, make sure to use our free test scoring at PowerScore Self Study Test Scoring.

      Next, I get the feeling you’ve seen this article before, but if not, it is key for those looking to increase their scores: What to Do Differently to Raise Your Score. In that article I give many trips on how to improve each section, as well as tests you can run to analyze your abilities and areas of weakness.

      This time you have coming up in May will be key, and the two month plan is a good base. Make sure to use that plan in conjunction with knowing your real areas of problem as described above, and use some of the topics in the last article I linked.

      Thanks!

      Reply

Leave a Reply Cancel reply

Your email address will not be published. Required fields are marked *

This site is protected by reCAPTCHA and the Google Privacy Policy and Terms of Service apply.

Attend a PowerScore Webinar!

Popular Posts

  • Podcast Episode 168: The 2025 US News Law School Rankings
  • Podcast Episode 167: April 2025 LSAT Recap
  • Podcast Episode 166: LSAT Faceoff: Dave and Jon Debate Five Common Test Concerns
  • Podcast Episode 165: February 2025 LSAT Recap
  • Podcast Episode 164: State of the LSAT Union: 2024 Recap and 2025 Preview

Categories

  • Pinterest
  • Facebook
  • YouTube
  • Twitter
Share this ArticleLike this article? Email it to a friend!

Email sent!